The product of 5 and the difference of 1 and a number.

Answers

Answer 1

Answer:

Simplify.

There's a product, so we are multiplying.

5 x...

The difference of one and a number. The number is a variable, since it could be any number.

1 - x

Let's combine them.

5 x (1 - x)

5(1 - x)

Hope this helped!

Mark this Brainliest, not that it matters.

Good luck :)


Related Questions

scarlett bought 10 chicken wings for $24.00. how much would it cost for 17 wings

Answers

First, divide.

24 / 10 = $2.40 per chicken wing

Second, multiply.

17 * 2.4 = $40.80

Best of Luck!

write an expression that represents the total amount of fencing needed to enclose the two fields. -how would you do this?-

Answers

Answer:

8x + 60

Step-by-step explanation:

4(x + 5) + 2(x - 10) + 2(x + 30)

= 4x + 20 + 2x - 20 + 2x + 60 (combine like terms)

= 8x + 60

What does x equal?
2(3x - 2) = 20

Answers

Answer:

4

Step-by-step explanation:

2(3x-2)=20

3x-2=10

3x=12

x=4

Answer:

x=4

Step-by-step explanation:

one-half the cube of m subtracted by 4 *

Answers

Answer:

[tex]\frac{m^3}{2} - 4[/tex]

Step-by-step explanation:

Required

Determine an expression for the given statement

The expression can be rewritten as:

one-half    *     the cube of m       subtracted by      4

Convert each statement to mathematical expressions:

One half = [tex]\frac{1}{2}[/tex]

The cube of m = [tex]m^3[/tex]

one-half    *     the cube of m = [tex]\frac{1}{2} * m^3[/tex]

At this stage, we have:

[tex]\frac{1}{2} * m^3[/tex] subtracted by 4

This gives:

[tex]\frac{1}{2} * m^3 - 4[/tex]

[tex]\frac{m^3}{2} - 4[/tex]

Hence, The corresponding expression is [tex]\frac{m^3}{2} - 4[/tex]

One-half the cube of m subtracted by 4 is [tex]0.5\sqrt[3]{m} -4[/tex]

Based on the information given, one-half the cube of m subtracted by 4 is expressed as:

[tex]0.5\sqrt[3]{m} -4[/tex]

Note that the cube of m is expressed as [tex]\sqrt[3]{m}[/tex]The half of the cube of m will be given as [tex]0.5\sqrt[3]{m}[/tex]

Subtracting 4 from the result will give the final expression as [tex]0.5\sqrt[3]{m} -4[/tex]

Learn more on expressions here: https://brainly.com/question/25968875

PLEASE HELP, ITS TIMED SO HURRY PLEASE>>>>>>>

Answers

Answer:

The answer is noncollinear

Step-by-step explanation:

Answer:

Non-collinear because the points dont line up to be colinnear. learned this today dead a** its not hard. pay attention

Step-by-step explanation:

QUESTION 11
(01.02)
Given the function f(x) = 3(x+2) - 4, solve for the inverse function when x = 2. (1 point)
O -4
Oo
O 4
08

Answers

Answer:

8

Step-by-step explanation:

3( 2 + 2 ) - 4

3( 4 ) - 4

12-4

8

Solve the inequation (2x+3x)4 using distributive method.

Answers

Answer:

First, distribute the 4.

2x times 4  = 8x

3x times 4 = 12x

then add/ combine like terms.

8x + 12x = 20x

Step-by-step explanation:

Answer:

20x

Step-by-step explanation:

distribute the 4 to the 2x and 3x: 4 * 2x= 8x and 4 * 3x= 12x

then add the x's together: 8x+12x=20x

so the answer is 20x

Point E is on line segment DF. Given DE = 2x, EF = 2x – 6, and
DF = 3x + 5, determine the numerical length of EF.

Answer: EF =

Answers

Answer:

EF is 16.  

Step-by-step explanation:

So we know that Point E is somewhere on Line DF. In other words, DE plus EF must equal DF. In an equation, this is:

[tex]DF=DE+EF[/tex]

We are given that DE is 2x, EF is 2x-6, and DF is 3x+5. So, substitute:

[tex](3x+5)=(2x)+(2x-6)[/tex]

Now, solve for x.

On the right, combine like terms:

[tex]3x+5=4x-6[/tex]

Subtract 4x from both sides:

[tex]-x+5=-6[/tex]

Subtract 5 from both sides:

[tex]-x=-11[/tex]

Divide both sides by -1:

[tex]x=11[/tex]

Now that we know x is 11, substitute this back into the equation for EF to find EF.

[tex]EF=2x-6[/tex]

Substitute 11 for x:

[tex]EF=2(11)-6[/tex]

Multiply:

[tex]EF=22-6[/tex]

Subtract:

[tex]EF=16[/tex]

And we're done!

10
Write the numbers in order from least to greatest (**HINT: You need to
make them all decimals or all fractions***): 5/1000, 0.003, 9/1000

Answers

Answer:

0.003, 5/1000, 9/1000

Step-by-step explanation:

0.003 is thousandths.

5/1000 is thousandths.

9/1000 is thousandths.

I converted them to fractions.

5/1000 is already a fraction

Same for 9/1000

0.003=3/1000

So 3/1000, 5/1000, 9/1000

Frank makes x dollars per week. His dad makes 4 times as much money as him each week. Each week their income adds up to 1.000 dollars. How much does frank earn each week? How much does franks dad earn each week?

Answers

Answer:

Frank earns $200.

His dad earns $800.

Step-by-step explanation:

Giving the following information:

His dad makes 4 times as much money as him each week. Each week their income adds up to 1.000 dollars.

We know that:

x= Franks income

4x= Dads income

y= total income

y= x + 4x

1,000= 5x

1,000/5=x

200=x

Frank earns $200.

His dad earns $800.

Do these two expressions represent equivalent expressions? Explain why or why not. 36 + 20 4(9 + 5)

Answers

Answer:

yes because 4 times 9 is equal to 36  and 4 times 5 is equal 20

Step-by-step explanation:

Yes, both the expressions ( 36 + 20 ) and 4 ( 9 + 5 ) are equivalent to each other.

What is an expression?

Expression in maths is defined as the collection of the numbers variables and functions by using signs like addition, subtraction, multiplication, and division.

The given expressions are solved as:-

First expression,

36 + 20 = 56

Second expression,

4( 9 + 5 ) = 36 + 20

               = 56

From the above calculations,

( 36 + 20 ) = 4 ( 9 + 5 )

Therefore, both the expressions ( 36 + 20 ) and 4 ( 9 + 5 ) are equivalent to each other.

To know more about Expression follow

https://brainly.com/question/723406

#SPJ2

8(4√7 - √18) I breed the answer fast please

Answers

Answer:

32[tex]\sqrt{7}[/tex] - 24[tex]\sqrt{2}[/tex]

Alternative form:

≈50.72292

Step-by-step explanation:

Step one,

Simplify the radical expression:

8(4√7 - √18)  

Still Stuck?  *( √18 Simplified is 3√2)*

Your answer should look like this:  8(4√7 - 3√2)

Step two,

Distribute 8 through the parentheses:

8(4√7 - 3√2)

Still Stuck?   *( Use distribuitave property to distribute 8 to (4) and (-3). You should get (32) and (-24)*

Your answer should look like this: 32√7 - 24√2

Solution,

32[tex]\sqrt{7}[/tex] - 24[tex]\sqrt{2}[/tex]

Alternative form:

≈50.72292

*For alternative form, plug the final answer into your calculator. Round it if necessary*

alternative form:50.72292

Which statement is true? 2.289 < 2.297 2.297 < 2.289 3.01 < 2.297 2.289 > 3.01​

Answers

Answer:

The first one, 2.2289<2.297

Step-by-step explanation:

Answer:

The answer is 2.2289<2.297 because if you look in the hundredths place there is a 9 in 2.297 but in 2.2289 there is an 2 and 9>2 so that is your answer.

solve for x: 4/x=5/10

Answers

simplify 5/10 to 1/2
1. 4/x = 1/2

multiply both sides by x
2. 4= 1/2 x

simply 1/2x to x/2
3. 4= x/2

multiply both sides by 2
4. 4x2= x


simplify 4 x 2 to 8
5. 8= x

switch sides
x= 8

ANSWER; x= 8

Answer:

x = 8

Step-by-step explanation:

4/x = 5/10

=> 4/x = 1/2

=> 4 x 2 = x

=> 8 = x

=> x = 8

PLEASE HELP ASAP Write the following equation in general form.
-x - 5y = -19

–x - 5y + 19 = 0
x + 5y - 19 = 0
x + 5y + 19 = 0

Answers

Answer:  -x - 5y + 19 = 0, choice A

All we're doing is adding 19 to both sides to get everything on the same side.

General form is Ax+By+C = 0. Some books use Ax+By = C which is just a variation of the same idea.

What is the greatest four-digit integer that meets the following three restrictions? 1- All of the digits are different. 2- The greatest digit is the sum of the other three digits. 3- The product of the four digits is divisible by 10 and not equal to zero.

Answers

I think that the greatest number that is all of these things is:9432

In a stationary store,pencils have one price and pens have another price.Two pencils and three pens cost 78 cents.However, 3 pencils and 2 pens cost 72 cents . How much does one pencil cost?

Answers

Answer:

12¢

Step-by-step explanation:

Let the unit price of a pencil be x and the unit price of a pen be y.

2x+3y=78¢

3x+2y=72¢

(2x)+3y=78¢

(3x)+2y=72¢

=2x=78¢-3y

=2x/2=78¢/2+3y/2

=x=36¢-1.5y

You then use the equation 3x+2y=72¢.

3(36¢-1.5y)+2y=72¢

=117¢-4.5y+2y=72¢

=117¢-72¢=4.5y-2y

=45¢=2.5y

=45¢/2.5=y

=18¢=y

So a pen costs 18¢.

You then use x=36¢-1.5y

x=36¢-1.5(18¢)

=x=36¢-27¢

=x=12¢

Therefore, a pencil costs 12¢.

Which angle refers to the same angle as ∠ D E F

Answers

Answer:

It is angle FED which is equal to angle DEF

1.5y-9.82=-9.37-10.1y-14.37 solve for y

Answers

Step-by-step explanation:

1.5y - 9.82 = -9.37 - 10.1y - 14.37

1.5y + 10.1y = 9.82 -9.37 - 14.37

11.6y = −13.92

y = (-13.92)/11.6

y = -1.2

why and how do you do this I am having a hard time

Answers

Answer:

Peter walks at a rate of 13/4 miles per hour.

Step-by-step explanation:

Peter walks at a rate of 13/4 miles per hour because every hour, represented by x, he walks 3.25 hours or 13/4. y is the total miles he walked so if he walks two hours the y would equal 6.5 or 26/4 and the x would equal 2.

What is 90% of 81?
X

Answers

Answer:72.9

Step-by-step explanation:

Answer:

72.9

Step-by-step explanation:

Fractions method:

[tex]81*\frac{90}{100}[/tex] because 90/100 is the same thing as 90%

That gives us 7290/100, from 81*90/100

Simplify: =729/10 = 72.9

Decimals method:

81*9 = 729, add in decimal and that's 72.9

Cross-multiplication:

90    100

?         81

This is read as 90 to 100, (as a ratio) so how many to 81? To solve, cross-multiply - 81*90=7290, then divide by 100 to get 72.9.

This is my first answer , hope it helped!

The dimensions of a rectangular prism are length is 2 1/4 feet width is 1 foot and height is 1 1/4 feet and the length of the sides of a small cube are 1/4 foot each. How many small cubs can be packed in the rectangular prism.

Answers

Answer:

  180

Step-by-step explanation:

2 1/4 ft = 9/4 ft, so is a length equivalent to 9 cubes.

1 ft = 4/4 ft, so is a length equivalent to 4 cubes.

1 1/4 ft = 5/4 ft, so is a length equivalent to 5 cubes.

Then the volume of the prism is ...

  V = LWH

  V = (9)(4)(5) = 180 . . . cubes

Given h(x)= 4x^2, find h(3)

Answers

Answer: 36

Explanation:

h(x) = 4x^2

Find h(3):

Simply plug x = 3 in the function:
h(3) = 4(3)^2
h(3) = 4(9)
h(3) = 36
You can make x 3 in this to help get the answer. h(3)= 4(3)^2. Using the order of operations, we do 3^2 first. The answer for that is 9. Then we do 9 times 4, which is 36. 36 is our final answer. Hope this helps!

What is the equation of a line with a slope of –2 that passes through the point (6, 8)?

Answers

Answer:

The answer is

y = - 2x + 20

Step-by-step explanation:

To find the equation of the line given a point and slope we use the formula

[tex]y - y_{1} = m(x - x _{1})[/tex]

where

m is the slope

( x1 , y1) is the point

From the question

The point is (6, 8) and

slope - 2

The equation of the line is

[tex]y - 8 = - 2(x - 6) \\ y - 8 = - 2x + 12 \\ y = - 2x + 12 + 8[/tex]

We have the final answer as

y = - 2x + 20

Hope this helps you

Answer:

C on edge

Step-by-step explanation:

just took the quiz

What is the ratio for 2 red paper clips to 6 blue paper clips? Write it as a fraction.

Answers

Answer:

2 over 6 as a fraction simplifies as 1 over three.

Step-by-step explanation:

If you are writing as a fraction, you put 2 over 6 and simplify. Your answer is 1 over 3

Answer:

2/6  2/6  2/6  2/6  2/6  2/6  2/6  2/6  2/6  2/6  2/6  2/6  2/6  2/6  2/6  2/6  2/6  2/6  2/6  2/6  2/6  2/6  2/6  2/6  2/6  2/6  2/6  2/6  2/6  2/6  2/6  2/6  2/6  2/6  2/6  2/6  2/6  2/6  2/6  2/6  2/6  2/6  2/6  2/6  2/6  2/6  2/6  2/6  2/6  2/6  2/6  2/6  2/6  2/6  2/6  2/6  2/6  2/6  2/6  2/6  2/6  2/6  2/6  2/6  2/6  2/6  2/6  2/6  2/6  2/6  2/6  2/6  2/6  2/6  2/6  2/6  2/6  2/6  2/6  2/6  2/6  2/6  2/6  2/6  2/6  2/6  2/6  2/6  2/6  2/6  2/6  2/6  2/6  2/6  2/6  2/6  2/6  2/6  2/6  2/6  2/6  2/6  

Step-by-step explanation:

its just 2/6

Put these into least to greatest

Answers

Answer:

Square root 8 over three, (square root of 4)/2, and pi/3.

Step-by-step explanation:

First of all, square the radicals to make solving this easier. Squarer root 8 over three will become 8/9, and square root 4 over 2 will now become 4/4, which is 1. Pi/3 is greater than 1, so we can now order this from least to greatest. 8/9 is the least, 1 is the second greatest value, and pi/3 is the greatest value.

Answer: 8squared/3 4squared/2 Pi/3

Step-by-step explanation:

8 squared = 2.8... = 2.8/3 (least)

4 squared = 2 = 2/2 = 1 (second least)

Pi over 3 = 3.14159.../3 = 1 .14159/3 (greatest)

Convert the given amount to the given unit.
27 yd; ft
27 yd = ft

Answers

Answer:

81 ft

Step-by-step explanation:

1 yard=3ft

27 x 3=81

So the answer is 81 ft

That would be 81 feet

Please help me! I am so bad at math. 7(6+5x)=3x-22

Answers

42+35x=3x–22
42+35x–3x=–22
42+32x=–22
32x=–22–42
32x=–64
x=–64/32
The answer is x= –2

Answer:

-2

Step-by-step explanation:

42+35x=3x-22

35x-3x=-22-42

32x=-64

therefore, x= -64/32

and through further simplification you will get

-2

hope this helped you ( ꈍᴗꈍ)

AG bisects ∢RAZ , and ∢RAG =. 8x+5 , and ∢ZAG = 4 x + 29 find the x .

Answers

Answer:

x = 6

Step-by-step explanation:

Given AG bisects ∠ RAZ , then

∠ RAG = ∠ ZAG , that is

8x + 5 = 4x + 29 ( subtract 4x from both sides )

4x + 5 = 29 ( subtract 5 from both sides )

4x = 24 ( divide both sides by 4 )

x = 6

Is 22 + 32 = 42 a true statement?

Answers

False because 22 + 32 =54

Answer:

No. 22+32=54

Step-by-step explanation:

Other Questions
1)Determine, in terms of unit vectors, the resultant of the five forces illustrated in the figure, Consider F1=20 N, F2= 12 N, F3=17 N. F4=15N and F5=9N 2)In the situation illustrated below, two blocks of Ma and Mb masses are suspended between two walls using ropes. If Ma=2kg, calculate the tension developed in the segments AB, BC and CD and the mass of block B for the equilibrium condition 3)In the figure below, the crane and its AB jib have together 390kg and center of mass in G1. If the 90kg BCD cage and the 80kg man have center of mass located in G2 and G3, respectively, (a) calculate the angle of inclination of the boom for which the crane is on the eminence of tumbling. (b) for which size the boom should be reduced (retracting its point B) so that it can be positioned horizontally (=0) and the crane does not tip over. Consider that the cage always stays in the horizontal position 4) A 1.7kg plate with center of mass in G is supported by a bar and three cables, as shown in the figure below. Determine the voltage developed in the AB, AC and DE cables and the reaction (there is only one) at point O for the equilibrium condition When planning for the future of a city.Urban planners would most likely consultA. physical geographersB. meteorologistsC. cartographersD. demographers Simplify the following: [tex]\frac{-1(-8x - 16)}{-2 -x} = \frac{-1(-16-8x)}{-2 -x}[/tex] Help please and thank you. During a trip Sara knits a scarf at the rate of 469 millimetre everyday if her trip lasts 8 days how much of the scar will she have knitted by the end of the trip The cable is 10 feet above the roadway at horizontal distances of about from the left support. Luke drove 260 miles in 4 hours. If he continued at the same rate how far would he travel in 18 hours? When the Better Builder Company completed a small construction job, they found that the following expenses had been incurred: labor, $672.25; gravel, $86.77; sand, $39.41; cement, $180.96; and bricks, $204.35. What total bill should they give the customer if they want to make a profit of $225 on the job? Find the value of x and y (3y+5) (2x+1) 85 A x= 42 y=26.3B x=47 y=30C x= 42 y=30D x=47 y= 26.3 When Juliet meets Romeo, what does she not know? A: She does not know he is poor. B: She does not know he is a Capulet. C: She does not know he is a Montague. D: She does not know he is angry with her. Which of them is important in endurance race physical strength or emotional strength? Which of the following changes in deciduous forests occurs in the spring?A. Trees regain their leaves.b. Trees block the flow of water and nutrients to their leaves.C.The leaves of trees begin to lose their chlorophyll.d. Trees live off of stored nutrients. they were doing up the parcels change into passive voice 1) Which equations are from the 2nd degree? * Tell me about what you know about "Rounding" the question can been seen in the picture Israel started to solve a radical equation in this way: square root of x plus 6 4 = xsquare root of x plus 6 4 + 4 = x + 4 square root of x plus 6 = x + 4 (square root of x plus 6)2 = (x + 4)2 x + 6 = x2 + 8x + 16 x + 6 6 = x2 + 8x + 16 6 x = x2 + 8x + 10 x x = x2 + 8x + 10 x 0 = x2 + 7x + 10 0 = (x + 2)(x + 5) x + 2 = 0 x + 5 = 0x + 2 2 = 0 2 x + 5 5 = 0 5 x = 2 x = 5 Solutions = 2, 5What error did Israel make? He subtracted 6 before subtracting x. He added 4 before squaring both sides. He factored x2 + 7x + 10 incorrectly. He did not check for extraneous solutions. Begin the story with "A girl waswalking home oneday If two events are correlated, what must be true? (1 point)O Both events have the same result.O Both events have the same cause.O The events tend to happen together.O One event caused the other. Which of the following was an effect of the Land Ordinance of 1785?It encouraged settlers to fully populate the thirteen states.It brought money to the nation through the sale of public lands.It allowed slavery to spread into the Northwest Territory.It limited the growth of schools and public education.